Last visit was: 24 Apr 2024, 16:54 It is currently 24 Apr 2024, 16:54

Close
GMAT Club Daily Prep
Thank you for using the timer - this advanced tool can estimate your performance and suggest more practice questions. We have subscribed you to Daily Prep Questions via email.

Customized
for You

we will pick new questions that match your level based on your Timer History

Track
Your Progress

every week, we’ll send you an estimated GMAT score based on your performance

Practice
Pays

we will pick new questions that match your level based on your Timer History
Not interested in getting valuable practice questions and articles delivered to your email? No problem, unsubscribe here.
Close
Request Expert Reply
Confirm Cancel
SORT BY:
Date
Tags:
Show Tags
Hide Tags
Board of Directors
Joined: 01 Sep 2010
Posts: 4383
Own Kudos [?]: 32869 [1]
Given Kudos: 4453
Send PM
Board of Directors
Joined: 01 Sep 2010
Posts: 4383
Own Kudos [?]: 32869 [1]
Given Kudos: 4453
Send PM
Board of Directors
Joined: 01 Sep 2010
Posts: 4383
Own Kudos [?]: 32869 [1]
Given Kudos: 4453
Send PM
Intern
Intern
Joined: 11 Jul 2012
Posts: 39
Own Kudos [?]: 205 [0]
Given Kudos: 25
GMAT 1: 650 Q49 V29
Send PM
Re: Hard CR questions involving proportion/numbers/statistics [#permalink]
carcass wrote:
avaneeshvyas wrote:
8. Twenty years ago, Balzania put in place regulations requiring operators of surface mines to pay for the reclamation of mined-out land. Since then, reclamation technology has not improved. Yet, the average reclamation cost for a surface coal mine being reclaimed today is only four dollars per ton of coal that the mine produced, less than half what it cost to reclaim surface mines in the years immediately after the regulations took effect.

Which of the following, if true, most helps to account for the drop in reclamation costs described?

A. Even after Balzania began requiring surface mine operators to pay reclamation costs, coal mines in Balzania continued to be less expensive to operate than coal mines in almost any other country.
B. In the twenty years since the regulations took effect, the use of coal as a fuel has declined from the level it was at in the previous twenty years.
C. Mine operators have generally ceased surface mining in the mountainous areas of Balzania because reclamation costs per ton of coal produced are particularly high for mines in such areas.
D. Even after Balzania began requiring surface mine operators to pay reclamation costs, surface mines continued to produce coal at a lower total cost than underground mines.
E. As compared to twenty years ago, a greater percentage of the coal mined in Balzania today comes from surface mines.
[Obscure] Spoiler:
OA C

Shortlisted my options to B and C, chose B because C didn't make sense to me.. :)...and also because I was able to justify B to myself by the logic that because if the coal consumption has been declining steadily over a period of 20 yrs it should then imply that no one is making money and there would be no scope of investing in a lost cause....hence the decline...actually I think there seems to be a flaw in my understanding of the term reclamation...what is being reclaimed and what cost are we talking about.....


here in my opinion you rushed the question. Look is silly your mistake :) B the use VS costs per ton.........the latter is what we really care about ;)


but the decline/increase in the usage will give us an idea about the consumption as well...... I think i get ur point though....C strengthens the argument more....
Intern
Intern
Joined: 11 Jul 2012
Posts: 39
Own Kudos [?]: 205 [0]
Given Kudos: 25
GMAT 1: 650 Q49 V29
Send PM
Re: Hard CR questions involving proportion/numbers/statistics [#permalink]
carcass wrote:
avaneeshvyas wrote:
Since I got the majority of the answers to the questions wrong... :cry: :cry: . I will try and initiate a discussion about the same.....

2. The violent crime rate (number of violent crimes per 1,000 residents) in Meadowbrook is 60 percent higher now than it was four years ago. The corresponding increase for Parkdale is only 10 percent. These figures support the conclusion that residents of Meadowbrook are more likely to become victims of violent crime than are residents of Parkdale.
The argument above is flawed because it fails to take into account

A. changes in the population density of both Parkdale and Meadowbrook over the past four years
B. how the rate of population growth in Meadowbrook over the past four years compares to the corresponding rate for Parkdale
C. the ratio of violent to nonviolent crimes committed during the past four years in Meadowbrook and Parkdale
D. the violent crime rates in Meadowbrook and Parkdale four years ago
E. how Meadowbrook’s expenditures for crime prevention over the past four years compare to Parkdale’s expenditures
[Obscure] Spoiler:
OA D

My question here would be....aren't we supposed to consider the population here.... I mean how can only taking the crime rates in both the cities be the deciding factor, as it may so happen that Parkdale's population has exceeded that of Meadowbrook and even with the the lower rate of violent crime rates it would have more crimes being committed(the number of crimes).....
P.S: I did zero onto D but rejected it because of the reasons mentioned here... and this has been happening to me with an alarming frequency off late... :oops: :cry:


The best strategy is

- First - Analysing carefully each sentence of the stimulus (you have already known that this is a flaw CR so, consequently you know you are searching something to weaken the argument or how in this case you need something that is the contrary of the assumpiton implied)

- Secondly - to start from wrong to right answer. This is your strategy, always

Now, right away you can eleiminate answer choices A C and E. Focus on B and D. B doens't have much sense but look at D for a moment. Her, basically you need something like the & change formula to be able to compare the two things \(\frac{New - original}{original}\)

Where is the original ?? D is the answer. You have to to do this process otherwise you always be in confusion. You have to have a systematic manner to handle the question.

Every GMAT question should be the same thing with a different mask NOT a challenge throughout. For this reason the best students are less tired at the end of the test: because work efficently ;)



My question is how can only the crime rates for both the cities be the deciding factors....since they themselves are dependent on the increase/decrease in the population..... and for crime rates you cannot possibly do \(\frac{New - original}{original}\) to conclude anything.....
Alum
Joined: 19 Mar 2012
Posts: 4341
Own Kudos [?]: 51447 [1]
Given Kudos: 2326
Location: United States (WA)
Concentration: Leadership, General Management
Schools: Ross '20 (M)
GMAT 1: 760 Q50 V42
GMAT 2: 740 Q49 V42 (Online)
GMAT 3: 760 Q50 V42 (Online)
GPA: 3.8
WE:Marketing (Non-Profit and Government)
Send PM
Re: Hard CR questions involving proportion/numbers/statistics [#permalink]
1
Kudos
Expert Reply
WOW! so much of discussions, i did not even have to participate!
Thanks so much Carcass!
User avatar
Manager
Manager
Joined: 21 Sep 2012
Posts: 181
Own Kudos [?]: 1329 [1]
Given Kudos: 63
Send PM
Re: Hard CR questions involving proportion/numbers/statistics [#permalink]
1
Kudos
avaneeshvyas wrote:

My question is how can only the crime rates for both the cities be the deciding factors....since they themselves are dependent on the increase/decrease in the population..... and for crime rates you cannot possibly do \(\frac{New - original}{original}\) to conclude anything.....


Example,
ratio of crimes M: P = 10: 20 ( four years ago)

M --> 1000 residents --> 10 (four years ago) --> 15 (now : 50% more)
P --> 1000 residents --> 20 (four years ago) --> 22 (now : 10% more)

This should make it very clear. D is the best answer.

If my post helped click +1 kudos.
Board of Directors
Joined: 01 Sep 2010
Posts: 4383
Own Kudos [?]: 32869 [0]
Given Kudos: 4453
Send PM
Re: Hard CR questions involving proportion/numbers/statistics [#permalink]
nelz007 wrote:
avaneeshvyas wrote:

My question is how can only the crime rates for both the cities be the deciding factors....since they themselves are dependent on the increase/decrease in the population..... and for crime rates you cannot possibly do \(\frac{New - original}{original}\) to conclude anything.....


Example,
ratio of crimes M: P = 10: 20 ( four years ago)

M --> 1000 residents --> 10 (four years ago) --> 15 (now : 50% more)
P --> 1000 residents --> 20 (four years ago) --> 22 (now : 10% more)

This should make it very clear. D is the best answer.

If my post helped click +1 kudos.



Here we go. Nice catch by you.

The worth strategy is to face this really difficult test with the most powerful tool: the reasoning. Do not be compartmentalized, trying to gather information from all areas of your knowledge.

For instance: to the core this question is solvable, comparing the two rates. but what about a sentence correction question where you have on the first portion of the sentence a statistic in your city, then in the second part of the sentence you MUST ave a second statistic to compare.

this is what I mean

Quote:
The unemployment rate in Esteria last month was 5.3%, compared to the rate in Burdistan.


WRONG

Quote:
The unemployment rate in Esteria last month was 5.3%, compared to a rate of 7% in Burdistan


CORRECT

Take away: try to handle a question from an odd angle, read each statement carefully. \(BE - FLEXIBLE\)
avatar
Intern
Intern
Joined: 07 May 2011
Posts: 15
Own Kudos [?]: 47 [0]
Given Kudos: 11
Send PM
Re: Hard CR questions involving proportion/numbers/statistics [#permalink]
Can anyone with verbal credentials that are equivalent to math credentials of Bunuel provide undebatable answers and explanations to these questions, please?

souvik101990 wrote:
Hello all.
This is the first of the collection of topic wise questions I am going to make here. This is a topic that hovers in the region where the quant part of the GMAT and the verbal part of the GMAT collide.
So try to solve the questions under 2.5 minutes and I will post the OAs in 2-3 days! You are free to google though!
Hopefully this set will help you clarify the doubts in this type of questions and leave no room for errors on the test day.

1. In the United States, of the people who moved from one state to another when they retired, the percentage who retired to Florida has decreased by three percentage points over the past ten years. Since many local businesses in Florida cater to retirees, this decline is likely to have a noticeably negative economic effect on these businesses.
Which of the following, if true, most seriously weakens the argument?


A. Florida attracts more people who move from one state to another when they retire than does any other state.
B. The number of people who move out of Florida to accept employment in other states has increased over the past ten years.
C. There are far more local businesses in Florida that cater to tourists than there are local businesses that cater to retirees.
D. The total number of people who retired and moved to another state for their retirement has increased significantly over the past ten years.
E. The number of people who left Florida when they retired to live in another state was greater last year than it was ten years ago.

2. The violent crime rate (number of violent crimes per 1,000 residents) in Meadowbrook is 60 percent higher now than it was four years ago. The corresponding increase for Parkdale is only 10 percent. These figures support the conclusion that residents of Meadowbrook are more likely to become victims of violent crime than are residents of Parkdale.
The argument above is flawed because it fails to take into account


A. changes in the population density of both Parkdale and Meadowbrook over the past four years
B. how the rate of population growth in Meadowbrook over the past four years compares to the corresponding rate for Parkdale
C. the ratio of violent to nonviolent crimes committed during the past four years in Meadowbrook and Parkdale
D. the violent crime rates in Meadowbrook and Parkdale four years ago
E. how Meadowbrook’s expenditures for crime prevention over the past four years compare to Parkdale’s expenditures

3. In Kravonia, the average salary for jobs requiring a college degree has always been higher than the average salary for jobs that do not require a degree. Over the last few years, the number of Kravonians enrolled in college has been growing steadily. Therefore, the number of Kravonians entering the job market who have at least the qualification of a college degree will eventuallty be significantly higher than it has been over the last few years.

Which of the following, if true, most seriously weakens the argument?


A. Kravonians with more than one college degree earn little more, on average, than do Kravonians with only one college degree
B. The average number of years Kravonian college students remain enrolled before completing a degree has been increasing over the past several of years.
C. Despite the increase in the number of Kravonians attending college, the percentage of the population attending college is lower in Kravonia than in most other countries.
D. In recent years, empoyers have been requiring college degrees for workers in jobs that were previously performed successfully by Kravonians who did not have college degrees.
E. For many years, employers in Kravonia hva ehad difficulty finding enough college graduates to fill the high-paying jobs that were available.3

4. Because postage rates are rising, Home Decorator magazine plans to maximize its profits by reducing by one half the number of issues it publishes each year. The quality of articles, the number of articles published per year, and the subscription price will not change. Market research shows that neither subscribers nor advertisers will be lost if the magazine's plan is instituted.

Which of the following, if true, provides the strongest evidence that the magazine's profits are likely to decline if the plan is instituted?


A. With the new postage rates, a typical issue under the proposed plan would cost about one-third more to mail than a typical current issue would.
B. The majority of the magazine's subscribers are less concerned about a possible reduction in the quantity of the magazine's articles than about a possible loss of the current high quality of its articles.
C. Many of the magazine's long-time subscribers would continue their subscriptions even if the subscription price were increased.
D. Most of the advertisers that purchase advertising space in the magazine will continue to spend the same amount on advertising per issue as they have in the past.
E. Production costs for the magazine are expected to remain stable.

5. Corporate Officer: Last year was an unusually poor one for our chemical division, which has traditionally contributed about 60 percent of the corporation’s profits. It is therefore encouraging that there is the following evidence that the pharmaceutical division is growing stronger: it contributed 45 percent of the corporation’s profits, up from 20 percent the previous year.
On the basis of the facts stated, which of the following is the best critique of the evidence presented above?


(A) The increase in the pharmaceutical division’s contribution to corporation profits could have resulted largely from the introduction of single, important new product.
(B) In multidivisional corporations that have pharmaceutical divisions, over half of the corporation’s profits usually come from the pharmaceuticals.
(C) The percentage of the corporation’s profits attributable to the pharmaceutical division could have increased even if that division’s performance had not improved.
(D) The information cited does not make it possible to determine whether the 20 percent share of profits cited was itself an improvement over the year before.
(E) The information cited does not make it possible to compare the performance of the chemical and pharmaceutical divisions in of the percent of total profits attributable to each.

6. Political Advertisement: Mayor Delmont’s critics complain about the jobs that were lost in the city under Delmont’s leadership. Yet the fact is that not only were more jobs created than were eliminated, but the average pay for these new jobs has been higher than the average pay for jobs citywide every year since Delmont took office. So there can be no question that throughout Delmont’s tenure the average paycheck in this city has been getting steadily bigger.
Which of the following, if true, most strengthens the argument in the advertisement?


A. The average pay for jobs created in the city during the past three years was higher than the average pay for jobs created in the city earlier in Mayor Delmont’s tenure.
B. Average pay in the city was at a ten-year low when Mayor Delmont took office.
C. Some of the jobs created in the city during Mayor Delmont’s tenure have in the meantime been eliminated again.
D. The average pay for jobs eliminated in the city during Mayor Delmont’s tenure has been roughly equal every year to the average pay for jobs citywide.
E. The average pay for jobs in the city is currently higher than it is for jobs in the suburbs surrounding the city.

7. Denoma, a major consumer-electronics maker, had a sizeable decline in sales revenue for its most recent fiscal year. This result appears surprising, because electronics retailers report that although their overall sales were considerably lower than in the previous year, their sales revenue from Denoma models actually grew, largely thanks to some innovative and popular models that Denoma introduced.
Which of the following, if true, does most to explain the apparently surprising result?

A. Because of the need to educate the public about its new models’ capabilities, Denoma’s advertising spending was higher than normal over the period.
B. For the period at issue, Denoma’s major competitors reported declines in revenue that were, in percentage terms, greater than Denoma’s.
C. A significant proportion of Denoma’s revenue comes from making components for other consumer-electronics manufacturers.
D. Unlike some of its major competitors, Denoma has no lines of business outside consumer electronics to provide revenue when retail sales of consumer electronics are weak.
E. During the period, consumer-electronics retailers sold remaining units of Denoma’s superseded models at prices that were deeply discounted from those models’ original prices.

8. Twenty years ago, Balzania put in place regulations requiring operators of surface mines to pay for the reclamation of mined-out land. Since then, reclamation technology has not improved. Yet, the average reclamation cost for a surface coal mine being reclaimed today is only four dollars per ton of coal that the mine produced, less than half what it cost to reclaim surface mines in the years immediately after the regulations took effect.

Which of the following, if true, most helps to account for the drop in reclamation costs described?


A. Even after Balzania began requiring surface mine operators to pay reclamation costs, coal mines in Balzania continued to be less expensive to operate than coal mines in almost any other country.
B. In the twenty years since the regulations took effect, the use of coal as a fuel has declined from the level it was at in the previous twenty years.
C. Mine operators have generally ceased surface mining in the mountainous areas of Balzania because reclamation costs per ton of coal produced are particularly high for mines in such areas.
D. Even after Balzania began requiring surface mine operators to pay reclamation costs, surface mines continued to produce coal at a lower total cost than underground mines.
E. As compared to twenty years ago, a greater percentage of the coal mined in Balzania today comes from surface mines.

9. For the first time in history, more televisions than people can be found in American households. According to recent research, the average household has 2.55 residents and contains 2.73 televisions. However, by employing such costly manufacturing processes as plasma technology and flat screens, televisions are becoming too expensive for the typical consumer. As a result, the average number of residents per household will again surpass the number of televisions.

Which of the following, if true, most strongly supports the argument above?


A. House parties at which numerous individuals gather to view popular television shows on one television set have increased three hundred percent during the past year.
B. More than one million legal immigrants enter the United States each year.
C. New devices such as video-enabled personal digital assistants and music players are increasingly purchased for use as a primary source of information and entertainment.
D. As new technologies become more commonplace, manufacturing and retail costs normally decline.
E. As a result of technological advances, new televisions are increasingly enabled with some features, such as Internet browsing, traditionally associated with other household devices.
avatar
Intern
Intern
Joined: 26 Sep 2012
Posts: 44
Own Kudos [?]: 35 [0]
Given Kudos: 21
GMAT 1: 680 Q49 V32
GMAT 2: 730 Q50 V40
GPA: 4
WE:Operations (Energy and Utilities)
Send PM
Re: Hard CR questions involving proportion/numbers/statistics [#permalink]
Got 6 out of 9 correct. Time : 18:03 minutes.

Thanks for the questions. Really challenging.
Retired Moderator
Joined: 27 Aug 2012
Posts: 1015
Own Kudos [?]: 4054 [0]
Given Kudos: 156
Send PM
Re: Hard CR questions involving proportion/numbers/statistics [#permalink]
Aha..! Some great qs. Souvik..

Look forward to seeing many of these of such difficulty level...
Alum
Joined: 19 Mar 2012
Posts: 4341
Own Kudos [?]: 51447 [0]
Given Kudos: 2326
Location: United States (WA)
Concentration: Leadership, General Management
Schools: Ross '20 (M)
GMAT 1: 760 Q50 V42
GMAT 2: 740 Q49 V42 (Online)
GMAT 3: 760 Q50 V42 (Online)
GPA: 3.8
WE:Marketing (Non-Profit and Government)
Send PM
Re: Hard CR questions involving proportion/numbers/statistics [#permalink]
Expert Reply
Retired Moderator
Joined: 27 Aug 2012
Posts: 1015
Own Kudos [?]: 4054 [0]
Given Kudos: 156
Send PM
Re: Hard CR questions involving proportion/numbers/statistics [#permalink]
Again Great Collections Souvik ...

Please keep posting more of these 700+ level qs. to keep us motivated.

Your sig. collections,going forward, seems to outnumber Bunuel's PS collections (which is already a benchmark in the Club)... :)
Retired Moderator
Joined: 27 Aug 2012
Posts: 1015
Own Kudos [?]: 4054 [0]
Given Kudos: 156
Send PM
Re: Hard CR questions involving proportion/numbers/statistics [#permalink]
souvik101990 wrote:
https://gmatclub.com/forum/modifier-set-143448.html

meaning-set-143449.html


Hey Souvik,
one suggestion mate..! is the "meaning-set-143449.html" already added to your sig ? if not please add the same (apparently I don't see in your sig.) as it's a great collection and would be of great help to the club.
8-)
Alum
Joined: 19 Mar 2012
Posts: 4341
Own Kudos [?]: 51447 [1]
Given Kudos: 2326
Location: United States (WA)
Concentration: Leadership, General Management
Schools: Ross '20 (M)
GMAT 1: 760 Q50 V42
GMAT 2: 740 Q49 V42 (Online)
GMAT 3: 760 Q50 V42 (Online)
GPA: 3.8
WE:Marketing (Non-Profit and Government)
Send PM
Re: Hard CR questions involving proportion/numbers/statistics [#permalink]
1
Kudos
Expert Reply
Added :-)
User avatar
Manager
Manager
Joined: 09 Nov 2012
Status:How easy it is?
Posts: 74
Own Kudos [?]: 408 [1]
Given Kudos: 174
Location: India
Concentration: Operations, General Management
GMAT 1: 650 Q50 V27
GMAT 2: 710 Q49 V37
GPA: 3.5
WE:Operations (Other)
Send PM
Re: Hard CR questions involving proportion/numbers/statistics [#permalink]
1
Kudos
avaneeshvyas wrote:
carcass wrote:
avaneeshvyas wrote:
Since I got the majority of the answers to the questions wrong... :cry: :cry: . I will try and initiate a discussion about the same.....

2. The violent crime rate (number of violent crimes per 1,000 residents) in Meadowbrook is 60 percent higher now than it was four years ago. The corresponding increase for Parkdale is only 10 percent. These figures support the conclusion that residents of Meadowbrook are more likely to become victims of violent crime than are residents of Parkdale.
The argument above is flawed because it fails to take into account

A. changes in the population density of both Parkdale and Meadowbrook over the past four years
B. how the rate of population growth in Meadowbrook over the past four years compares to the corresponding rate for Parkdale
C. the ratio of violent to nonviolent crimes committed during the past four years in Meadowbrook and Parkdale
D. the violent crime rates in Meadowbrook and Parkdale four years ago
E. how Meadowbrook’s expenditures for crime prevention over the past four years compare to Parkdale’s expenditures
[Obscure] Spoiler:
OA D

My question here would be....aren't we supposed to consider the population here.... I mean how can only taking the crime rates in both the cities be the deciding factor, as it may so happen that Parkdale's population has exceeded that of Meadowbrook and even with the the lower rate of violent crime rates it would have more crimes being committed(the number of crimes).....
P.S: I did zero onto D but rejected it because of the reasons mentioned here... and this has been happening to me with an alarming frequency off late... :oops: :cry:


The best strategy is

- First - Analysing carefully each sentence of the stimulus (you have already known that this is a flaw CR so, consequently you know you are searching something to weaken the argument or how in this case you need something that is the contrary of the assumpiton implied)

- Secondly - to start from wrong to right answer. This is your strategy, always

Now, right away you can eleiminate answer choices A C and E. Focus on B and D. B doens't have much sense but look at D for a moment. Her, basically you need something like the & change formula to be able to compare the two things \(\frac{New - original}{original}\)

Where is the original ?? D is the answer. You have to to do this process otherwise you always be in confusion. You have to have a systematic manner to handle the question.

Every GMAT question should be the same thing with a different mask NOT a challenge throughout. For this reason the best students are less tired at the end of the test: because work efficently ;)



My question is how can only the crime rates for both the cities be the deciding factors....since they themselves are dependent on the increase/decrease in the population..... and for crime rates you cannot possibly do \(\frac{New - original}{original}\) to conclude anything.....

I have the same doubt as aveenashvyas, seeing the explanations I do not find a clarification of the doubt that crime rate (which is defined as number or crimes per 1000 residents) is dependent on population! How can it not be dependent and how can the rate of growth of population be not relevant to crime rate?
MBA Section Director
Joined: 22 Feb 2012
Affiliations: GMAT Club
Posts: 8701
Own Kudos [?]: 10011 [1]
Given Kudos: 4542
Test: Test
Send PM
Re: Hard CR questions involving proportion/numbers/statistics [#permalink]
1
Kudos
Expert Reply
souvik101990 wrote:
1. In the United States, of the people who moved from one state to another when they retired, the percentage who retired to Florida has decreased by three percentage points over the past ten years. Since many local businesses in Florida cater to retirees, this decline is likely to have a noticeably negative economic effect on these businesses.
Which of the following, if true, most seriously weakens the argument?


A. Florida attracts more people who move from one state to another when they retire than does any other state.
B. The number of people who move out of Florida to accept employment in other states has increased over the past ten years.
C. There are far more local businesses in Florida that cater to tourists than there are local businesses that cater to retirees.
D. The total number of people who retired and moved to another state for their retirement has increased significantly over the past ten years.
E. The number of people who left Florida when they retired to live in another state was greater last year than it was ten years ago.



Answer should be D

Premise :- of the people who retire and move to another state, percentage of people shifted to Florida decreased by 3 percentage points.

Conclusion :- Businesses that cater to retirees are likely to affect by this decline.

Analysis :- Since the businesses will only get affected when actual number of customers decline. That means when actual number of retirees that shifts to Florida decline. If this doesn't happen then conclusion will not hold true.
Suppose in 2001 in the US 100 people moved after retirement. of which 50% i.e. 50 shifted to Florida and rest 50 shifted to other states. While in 2010, total of 200 people moved after retirement of which 47% i.e. 94 shifted to Florida and 53% i.e. 106 shifted in other states.
In this scenario actual number of retirees shifted to Florida increased and hence businesses that caters retirees will not affect at all.
So if number of people who moves after retirement in the whole nation increase then conclusion will be undermined.

A) Even though Florida attracts more people than does any other state, if the number of people who move after retirement stays same over the period of 10 years then actual number of people moves to Florida after retirement will decline. So this choice does not undermine the conclusion. INCORRECT

B) Stimulus talks about businesses that Cater demands of Retirees and not about those serves general population. INCORRECT

C) Same as above. Stimulus is concerned only about businesses that serves retirees. INCORRECT

D) This choice says Actual Number of people who retired and moved in other states increased SUBSTANTIALLY. This is in accordance with our analysis. CORRECT

E) This choice strengthens the conclusion. It says actual number of retirees living in Florida has declined in 10 years. INCORRECT
MBA Section Director
Joined: 22 Feb 2012
Affiliations: GMAT Club
Posts: 8701
Own Kudos [?]: 10011 [0]
Given Kudos: 4542
Test: Test
Send PM
Re: Hard CR questions involving proportion/numbers/statistics [#permalink]
Expert Reply
souvik101990 wrote:
2. The violent crime rate (number of violent crimes per 1,000 residents) in Meadowbrook is 60 percent higher now than it was four years ago. The corresponding increase for Parkdale is only 10 percent. These figures support the conclusion that residents of Meadowbrook are more likely to become victims of violent crime than are residents of Parkdale.
The argument above is flawed because it fails to take into account


A. changes in the population density of both Parkdale and Meadowbrook over the past four years
B. how the rate of population growth in Meadowbrook over the past four years compares to the corresponding rate for Parkdale
C. the ratio of violent to nonviolent crimes committed during the past four years in Meadowbrook and Parkdale
D. the violent crime rates in Meadowbrook and Parkdale four years ago
E. how Meadowbrook’s expenditures for crime prevention over the past four years compare to Parkdale’s expenditures



Answer Should be D

Analysis :-
Suppose 4 years back the rate of violent crimes in Meadowbrook was 10 per 1000. After 4 years there was 60% increase in the rate of violent crimes in Meadowbrook. So it must be 16 per 1000.

Let the initial rate of violent crimes in Parkdale be 100 per 1000. In 4 years there was 10% increase. So now it must be 110 per 1000.

In this case residents of Parkdale are still likely to become victims of violent crimes and not the residents of Meadowbrook.

Argument is flawed because it falsely assumes that Initial rate of violent crimes in Meadowbrook was equal to or greater than that in Parkdale.


A) Since the RATE of violent crimes has increased, In the event of population increase or its density increase, number of crimes is sure to increase subsequently. This indeed strengthen the argument and does not reveals the flaw in the argument. INCORRECT

B) Different rates of population growth would not have any effect on Probability of becoming a victim of violent crime as long as rates of crime are also increasing. INCORRECT

C) Non-violent crimes are not part of the discussion. INCORRECT

D) It is necessary to know the initial crime rates before drawing the given conclusion. CORRECT

E) Out of Scope. INCORRECT
MBA Section Director
Joined: 22 Feb 2012
Affiliations: GMAT Club
Posts: 8701
Own Kudos [?]: 10011 [1]
Given Kudos: 4542
Test: Test
Send PM
Re: Hard CR questions involving proportion/numbers/statistics [#permalink]
1
Bookmarks
Expert Reply
souvik101990 wrote:
3. In Kravonia, the average salary for jobs requiring a college degree has always been higher than the average salary for jobs that do not require a degree. Over the last few years, the number of Kravonians enrolled in college has been growing steadily. Therefore, the number of Kravonians entering the job market who have at least the qualification of a college degree will eventuallty be significantly higher than it has been over the last few years.

Which of the following, if true, most seriously weakens the argument?


A. Kravonians with more than one college degree earn little more, on average, than do Kravonians with only one college degree
B. The average number of years Kravonian college students remain enrolled before completing a degree has been increasing over the past several of years.
C. Despite the increase in the number of Kravonians attending college, the percentage of the population attending college is lower in Kravonia than in most other countries.
D. In recent years, empoyers have been requiring college degrees for workers in jobs that were previously performed successfully by Kravonians who did not have college degrees.
E. For many years, employers in Kravonia hva ehad difficulty finding enough college graduates to fill the high-paying jobs that were available.3



Answer Should be B


The First sentence "the average salary for jobs………..do not require a degree" is not directly related to the conclusion. We have to only consider the remaining part.

Premise :- over the last few years, number of kravonions enrolled in college has been growing steadily.

Conclusion :- Therefore more people having atleast college degrees will enter in the job market in future.

Weakening scenario :- Though more people entered the college, if they do not finish their degrees then they will not be able to enter in the job market and in that case conclusion will not hold true

A) We do not have evidance to assume that people are turning towards college degrees because of higher salaries offered to degree holders. INCORRECT

B) This choice states that number of people finishing their degrees will not increase as average number of years remain enrolled for degrees has been growing. This is similar to our pre-thinking. CORRECT

C) Percentage of population attending college of Kravonia is compared with that of other countries. Out of Scope. INCORRECT

D) This choice strengthens the conclusion. It says that requirement of college degree holders has been increasing. INCORRECT

E) This choice says that since years there was shortge of college graduates. This partially explains why there is high requirements of college graduates in the market. But it does not comment on whether Number of college graduates are increasing. INCORRECT.
MBA Section Director
Joined: 22 Feb 2012
Affiliations: GMAT Club
Posts: 8701
Own Kudos [?]: 10011 [0]
Given Kudos: 4542
Test: Test
Send PM
Re: Hard CR questions involving proportion/numbers/statistics [#permalink]
Expert Reply
souvik101990 wrote:
4. Because postage rates are rising, Home Decorator magazine plans to maximize its profits by reducing by one half the number of issues it publishes each year. The quality of articles, the number of articles published per year, and the subscription price will not change. Market research shows that neither subscribers nor advertisers will be lost if the magazine's plan is instituted.

Which of the following, if true, provides the strongest evidence that the magazine's profits are likely to decline if the plan is instituted?


A. With the new postage rates, a typical issue under the proposed plan would cost about one-third more to mail than a typical current issue would.
B. The majority of the magazine's subscribers are less concerned about a possible reduction in the quantity of the magazine's articles than about a possible loss of the current high quality of its articles.
C. Many of the magazine's long-time subscribers would continue their subscriptions even if the subscription price were increased.
D. Most of the advertisers that purchase advertising space in the magazine will continue to spend the same amount on advertising per issue as they have in the past.
E. Production costs for the magazine are expected to remain stable.



Answer should be D

Premise 1 :- Home Decorator magazine plans to reduce number of issues by 50% to maximize its profits to cope up with rising Postage rates. While doing so magazine would not reduce the number of articles it publishes every year and will not compromise with the quality of articles. Subscription Price will also remain same.

Premise 2 :- Neither subscribers nor advertisers will be lost if the plan is instituted.

Assumption 1 :- Advertisers will succeed in reaching their targeted customers to the extent they were expecting with the reduced issues.

Assumption 2 :- Magazines current revenue (Subscription + Advertising Amount) will not decrease with the reduction in issues.

Assumption 3 :- The money saved from reduction in the number of issues published will be greater than that spent on additional postage charge, if any, for new larger sized magazine.

Analysis :- To Prove that magazine's profit will decline we need to attack any one of the 2nd and 3rd assumption. If Magazine loses some of its revenue (if this is the case, then loss will certainly come from Advertising fee since it has already stated in the premise that subscription will be unaffected) or its new issue costs substantially more to post than current issue then the magazine's profits will decline.

A) Let the current No of issue be 100 and current postage rate be 10 per issue. Then Current Postage Expenditure would be 1000. According to this choice new postage rate will be 13.33 per issue. Number of issues will be 50. Hence Total Postage expenditure would be around 650. Magazine still can increase its profit atleast by 350. So this choice is INCORRECT

B) As per this choice, majority of subscribers are concerned about possible loss of current high quality of magazine's articles and It has already cleared in the premise 1 that no compromise will be done with the quality of articles. Hence INCORRECT

C) This choice supports the Assumption 1 that subscription will not be affected. INCORRECT

D) If the advertisers continue to pay with the same rate per issue then the revenue from advertising will be halved, decreasing the overall revenue of the magazine, because now number of issues published will be fewer. This choice proves that the profits would decline. CORRECT

E) Not sufficient. INCORRECT
GMAT Club Bot
Re: Hard CR questions involving proportion/numbers/statistics [#permalink]
   1   2   3   
Moderators:
GMAT Club Verbal Expert
6920 posts
GMAT Club Verbal Expert
238 posts
CR Forum Moderator
832 posts

Powered by phpBB © phpBB Group | Emoji artwork provided by EmojiOne